LSAT and Law School Admissions Forum

Get expert LSAT preparation and law school admissions advice from PowerScore Test Preparation.

 Administrator
PowerScore Staff
  • PowerScore Staff
  • Posts: 8916
  • Joined: Feb 02, 2011
|
#98369
Complete Question Explanation

Must Be True, Principle. The correct answer choice is (B).

Answer choice (A):

Answer choice (B): This is the correct answer choice.

Answer choice (C):

Answer choice (D):

Answer choice (E):

This explanation is still in progress. Please post any questions below!
User avatar
 katnyc
  • Posts: 35
  • Joined: Dec 22, 2020
|
#100041
Hello,
I was between choices B and C. I got it right but I couldnt find an explanation to fully cross out C compared to the others. Any help would be appreciated.
 Rachael Wilkenfeld
PowerScore Staff
  • PowerScore Staff
  • Posts: 1358
  • Joined: Dec 15, 2011
|
#100648
Hi kat,

Answer choice (C) talks about effort here, a concept that was not discussed at all in the stimulus. We don't know at all about the effort required for tasks. More efficient is not the same as having less effort required. This is a must be true question, so the correct answer choice must be based on information from the stimulus.

Hope that helps!
User avatar
 FreeTim
  • Posts: 2
  • Joined: Nov 18, 2023
|
#104028
What is the difference between answer B and D?

I answered D, because it was slightly more specific. What was my mistake and how can I learn from it?
Thank you
 Luke Haqq
PowerScore Staff
  • PowerScore Staff
  • Posts: 747
  • Joined: Apr 26, 2012
|
#104042
Hi FreeTim!

One problem with answer choice (D) is the "will likely" language. Another problem is that the answer choice is not limited just to industrial societies.

This is a must be true question, but there's nothing in the stimulus stating that an increase in workers' standards of living will generally decrease how much workers value their jobs. Rather the stimulus is just limited to industrialized societies that have been shaped by Taylor's efficiency movement. In addition, the stimulus doesn't directly say that workers will value their work less--only that their work will lose some of its intrinsic worth.

Answer (B) doesn't have these problems. The answer choice is limited to industry. It also uses the word "can," which is weaker than "will likely," making it easier to establish as true. Finally, it doesn't make a claim about how much workers "value their jobs overall" (as in (D)) but rather just notes that some developments in industry that benefit workers can make the work less rewarding generally "in other respects" (i.e., with respect to the creativity and talent it used to require).

Get the most out of your LSAT Prep Plus subscription.

Analyze and track your performance with our Testing and Analytics Package.